Math, asked by Anonymous, 3 months ago

A continuous random variable X has a p.d.f. f(x)=3x2 , 0≤x≤1. Find a and b such that (i) P(X≤a)=P(X>a), and P(X>b)=0.05.​

Answers

Answered by MaheswariS
55

\textbf{Given:}

\textsf{Probability density function is}

\mathsf{f(x)=3x^2,\;\;0\leq\,x\leq\;1}

\mathsf{P(X\leq\,a)=P(X\,>\,a)\;\;and}

\mathsf{P(X\,>\,b)=0.05}

\textbf{To find:}

\textsf{Values of a and b}

\textbf{Solution:}

\textsf{Consider,}

\mathsf{P(X\leq\,a)=P(X\,>\,a)}

\mathsf{P(X\leq\,a)=1-P(X\leq\,a)}

\mathsf{2\;P(X\leq\,a)=1}

\mathsf{2\;\int\limits^{a}_0\,f(x)\,dx=1}

\mathsf{2\;\int\limits^{a}_0\,3x^2\,dx=1}

\mathsf{6\;\int\limits^{a}_0\,x^2\,dx=1}

\mathsf{6\left(\dfrac{x^3}{3}\right)^a_0=1}

\mathsf{2(x^3)^a_0=1}

\mathsf{(x^3)^a_0=\dfrac{1}{2}}

\mathsf{a^3-0=\dfrac{1}{2}}

\mathsf{a^3=0.5}

\implies\boxed{\mathsf{a=(0.5)^\frac{1}{3}}}

\mathsf{Consider,}

\mathsf{P(X\,>\,b)=0.05}

\mathsf{\int\limits^1_b\,f(x)\,dx=0.05}

\mathsf{\int\limits^1_b\,3x^2\,dx=0.05}

\mathsf{3\left(\dfrac{x^3}{3}\right)\limits^1_b=0.05}

\mathsf{(x^3)\limits^1_b=0.05}

\mathsf{1^3-b^3=0.05}

\mathsf{b^3=1-0.05}

\mathsf{b^3=0.95}

\implies\boxed{\mathsf{b=(0.95)^\frac{1}{3}}}

Answered by Sai7396
1

Answer:

gud

Step-by-step explanation:

Similar questions